Question 3 of 10
True or false? In a two-column proof, the right column states your reasons.
A. True
OB. False
SUBMIT

Answers

Answer 1

Answer:

A- True

Step-by-step explanation:

If you search a picture of the graph then you will see it as well!!! Hope this helps!!!!


Related Questions

-4(-w-10)=
i need help don’t know what the answer is

Answers

Answer:

[tex]\boxed{4w+40}[/tex]

Step-by-step explanation:

Use the distributive property to evaluate the equation.

-4(-w - 10)

4w + 40

-4 * -w = 4w because two negatives cancel to make a positive value.

-4 * -10 = 40 for the same reason.

Which of the c-values satisfy the following inequality? 2>c/3

Answers

Answer:

[tex]\Large \boxed{{c<6}}[/tex]

Step-by-step explanation:

2>c/3

Multiply both sides of the inequality by 3.

2(3)>c/3(3)

6>c

Switch sides.

c<6

●✴︎✴︎✴︎✴︎✴︎✴︎✴︎✴︎❀✴︎✴︎✴︎✴︎✴︎✴︎✴︎✴︎✴︎●

Hi my lil bunny!

❧⎯⎯⎯⎯⎯⎯⎯⎯⎯⎯⎯⎯⎯⎯⎯⎯⎯⎯⎯⎯⎯⎯⎯⎯⎯⎯⎯⎯⎯⎯⎯⎯⎯⎯⎯⎯⎯⎯☙

Let's solve your inequality step-by-step.

[tex]2 > \frac{c}{3}[/tex]

Step 1: Simplify both sides of the inequality.

[tex]2 > \frac{1}{3}c[/tex]

Step 2: Flip the equation.

[tex]\frac{1}{3}c < 2[/tex]

Step 3: Multiply both sides by 3.

[tex]3 * (\frac{1}{3} c) < ( 3) * (2)\\c < 6[/tex]

Answer : [tex]\boxed {c < 6}[/tex]

❧⎯⎯⎯⎯⎯⎯⎯⎯⎯⎯⎯⎯⎯⎯⎯⎯⎯⎯⎯⎯⎯⎯⎯⎯⎯⎯⎯⎯⎯⎯⎯⎯⎯⎯⎯⎯⎯⎯☙

●✴︎✴︎✴︎✴︎✴︎✴︎✴︎✴︎❀✴︎✴︎✴︎✴︎✴︎✴︎✴︎✴︎✴︎●

Have a great day/night!

❀*May*❀

the school bought a sandbox that measures 50 meters long 25 meters wide and 5 meters tall how many cubic meters if sand would you need to buy if each cubic meter of sand cost $1.50 how much money would it cost to fill the sandbox

Answers

Answer:

Cost of sandbox = $9,375

Step-by-step explanation:

Given:

Height of sandbox = 5 m

Length of sandbox = 50 m

Width of sandbox = 25 m

Cost of 1 cubic meter = $1.50

Find:

Cost of sandbox

Computation:

Volume of sandbox = (50)(25)(5)

Volume of sandbox = 6,250 m³

Cost of sandbox = 6,250 × $1.50

Cost of sandbox = $9,375

10 points :) Graph this for me :P

Answers

Answer:

-2≤x≤2   f(x)=[x+3]

first  the sign is ≤ it means the point is solid point and the interval is x+3

Given that the quadrilateral is a parallelogram, m∠S = + 19 and m ∠T = 8x - 4, what is m∠Q?

Answers

Answer:

32

Step-by-step explanation:

Brainlist plz

Answer:

[tex]\Large \boxed{161\°}[/tex]

Step-by-step explanation:

Adjacent angles in a parallelogram add up to 180 degrees.

m∠Q and m∠S add up to 180 degrees.

m∠Q + m∠S = 180

m∠Q + 19 = 180

Subtract 19 from both sides.

m∠Q + 19 - 19 = 180 - 19

m∠Q = 161

A chocolate company has a new candy bar in the shape of a prism whose base is a 1-inch equilateral triangle and whose sides are rectangles that measure 1 inch by 2 inches. These prisms will be packed in a box that has a regular hexagonal base with 2-inch edges, and rectangular sides that are 6 inches tall. How many candy bars fit in such a box

Answers

BASE AREA OF THE BOX:

Regular hexagon with 2 inch edges contains 180*(6-2) = 180*4 = 720 degrees
That's 120 degrees in each of the corner pockets.
Drawing line segments from the center to each vertex creates 6 congruent isosceles triangles.
Selecting any one of these six triangles and dropping the height from the center to the edge
creates a 30-60-90 right triangle with height equal to the square root of 3 (rad3) and base measure 1.
So the area of each of these triangles is the square root of 3 , or in slang terms "rad 3", which means
the base area is 6*rad3.

The volume of the box is therefore 36*rad3

BASE AREA OF THE CANDY:

Dropping the height from one vertex to the opposite side of this equilateral triangle
creates 2 right triangles with base measure 1/2 and hypotneuse 1. So the height
must be rad3/2. The area of the entire equilateral triangle is rad3/4, which is the base area.

The volume of the candy is rad3/2


36*rad3 divided by rad3/2 =

36 rad3 times 2/rad3 <--- KFC : keep , change, flip

36 * 2 <-- rad3 cancels out

72 candies will fill the box.

What is negative sqrt 64?

Answers

Answer:

8i. In real numbers only, this isnt possible, but if immagenary numbers are allowed then 8i is your answer          

Samuel has $20 in his savings account before he makes a deposit of $160 after two weeks he withdraws $160 how did Samuel savings account balance change

Answers

Samuel’s saving account changed from 20 to 180 to back to 20

In each of the following pairs of numbers, state which whole number is on the left of the other number on the number line. Also write them with appropriate sign (>,<) between them (a) 530, 503 (b) 370, 307 (c) 98765, 56789 (d) 10023001

Answers

D) in the question

1002, 3001

Answer:

a) 503 b) 307 c) 56789 D) 1002

Step-by-step explanation:

Numbering starts from the left hand side and are arranged in ascending order.

A) (530, 503)

503 < 530

503 is lesser than 530 and as such it is encountered first when numbers are written, hence 503 will appear on the left side of the number line when both are written.

B.) (370, 307)

370 > 307

307 comes before 370, therefore it will appear on the left side of 370 when illustrated on a number line.

C.) (98765, 56789)

98765 > 56789

56789 will appear on the leftside of 98765 when both number are illustrated on a number line.

D.) 1002 < 3001

1002 will appear on the leftside of 98765 when both number are illustrated on a number line.

Plz help me this is probably easy but im just not seeing it plz help me ASAP

Answers

Answer: is attatched to the photo I sent

Step-by-step explanation:

I admit, this is tricky, just remember to think about it as where the shapes are the most similar. I'm not exactly sure how to explain it, but where it reflects like a mirror is the "copy"

The example I'm thinking of attached too, but these over lap so its diffucult

A certain mixture of paint contains 5 parts white paint for every 4 parts blue paint. If a can of paint contains 75 ounces of white paint, how many ounces of blue paint are in the can?

Answers

Answer:

i think 60 parts of blue paint are in the can.

Step-by-step explanation:

ratio should be equal in both cases

therefore,let blue part in second case be x(suppose)

5÷4=75÷x

by equating this we will be able to identify the value of x

and the value of x comes 60 which is the required answer....

hope this will help you..

i try my best to give correct answer..

if i am mistake, i am sorry for that...

Write each of the following in the simplest exponential form please help

Answers

Answer:

Step-by-step explanation:

[tex]a^{m}*a^{n} =a^{m+n}\\\\\\2^{3}*2^{5} = 2^{3+5} =2^{8}\\\\\\3^{2}*3^{4}=3^{2+4} = 3^{6}\\\\\\a^{2}*a^{5}=a^{2+5}=a^{7}[/tex]

A triangle has one side that lies along the line y=1/4x and another that lies along the line y=-1/4x. Which of the following points could be a vertex of the triangle?

Answers

Answer:

We know that our triangle has one side along the line:

y = (1/4)*x

And other side along the line:

y = -(1/4)*x.

Now, we want to find the vertex.

And we know that the vertex is the point where the two sides conect, so the vertex must be a common point of both lines.

Then we have:

y = (1/4)*x = -(1/4)*x

x = -x

The only solution to that equation is x = 0.

now we evaluate our lines in x = 0 and get:

y = (1/4)*0 = 0

y = -(1/4)*0 = 0

Then the lines intersect in the point (0, 0)

Then the vertex must be in the point (0, 0)

A fish jumps out of the water at a speed of 12 feet per second. The height y (in feet) of the fish above the surface of the water is represented by the equation y=-16x^2+12x, where x is the time (in seconds) since the jump began. The fish reaches its highest point above the surface of the water after 0.375 seconds. How far above the surface is the fish at this time?

Answers

Answer:

The fish is 2.25 ft above the surface at 0.375 seconds

Step-by-step explanation:

Given:

y=-16x^2+12x

x=0.375 seconds

Substitute x=0.375 into the equation

y=-16x^2+12x

= -16(0.375)^2 +12(0.375

= -16(0.140625) + 4.5

= -2.25 + 4.5

= 2.25

y=2.25 ft

The fish is 2.25 ft above the surface at 0.375 seconds

Pls answer these McQ to be the brainliest

Answers

Step-by-step explanation:

1. c

2. a

3. c

4. c

5. d

6. b

7. a

8. b

9. c

Help pleasee!!! Tyyyyy

Answers

Answer:

G. 7

Explanation:

The longest side must be less than the sum of the two shorter sides.

The difference between the longest and shortest sides must be less than the middle side.

l+a = 11

l < (a + 4) ... l < (11 - l + 4)

... 2 l < 15 ... l <  7.5

--Variables

l = 7 (Longest Side/Answer)

a = 4 (Other Missing Length)

m = 4 (Side Given)

Jack is building a square garden. Each side length measures 777 meters. Jack multiplies 7\times77×77, times, 7 to find the amount of space in his garden is equal to 494949 square meters. Which measurement does 494949 square meters represent?

Answers

Answer:

49 square meters represent area of the square garden

Step-by-step explanation:

Each side length=7 meters

He multiplied 7 × 7 times to find the amount of space

=49 square meters

Jack is trying to measure the area of his square garden

Area of the square garden = length^2

=Length × length

Recall,

Length=7 meters

Area of the square garden= 7 meters × 7 meters

=49 square meters

interpret the parts of the expression 9x + 4y – 5. Rewrite the expression as a sum: _______________

Answers

Answer:

9x + 4y +(-5)

Step-by-step explanation:

Given

9x + 4y - 5

Required

Interpret

Write as a sum

The parts of an expression can be interpreted in the following ways; Terms, Variables, Constant, Coefficient, etc.

The terms are the expression being added together and they are 9x, 4y and -5

The variables are the represented with alphabets they change in values; the two variables in the given question are x and y

Constant are numbers standing alone; This is 5

Coefficient are numbers in front of variables; In this case, the coefficient are 9 and 4

Writing 9x + 4y - 5 as a sum

The -5 can be written as +(-5); So, we have

9x + 4y +(-5)

What is the approximate volume of the whole sphere? Use pie=3.14 and round to the nearest whole cubic unit.
a. 42 cubic units
b. 126 cubic units
c. 4,187 cubic units
d. 73,385 cubic units

if you could awnser this as soon as possible that would be great :)​

Answers

Answer:

c. 4187 cubic units

Step-by-step explanation:

V = 4/3 π r³

= 4/3(3.14)(10)³

= 4186.6666666

The amount of juice in a container is

normally distributed with a mean of

70 ounces and a standard deviation of

0.5 ounce. What is the probability that a

randomly selected container has more

than 70.5 ounces of juice?

Answers

Answer:

0.15866

Step-by-step explanation:

To solve for the above question, we use z score formula

z = (x - μ) / σ, where

x is the raw score

μ is the population mean

σ is the population standard deviation.

From the question,

x is the raw score = 70.5

μ is the population mean = 70

σ is the population standard deviation = 0.5

z = 70.5 - 70/ 0.5

z= 0.5/0.5

z = 1

We used the z score table to find the probability of z = 1

P( x = z) = P(x = 70.5)

= P( z = 1) = 0.84134

Hence, the probability that a randomly selected container has more than 70.5 ounces of juice is calculated as

P(x>70.5) = 1 - P(x =70.5)

P(x>70.5) = 1 - 0.84134 = 0.15866

Therefore, the probability that a randomly selected container has more than 70.5 ounces of juice is 0.15866.

A national sampling of cookie preferences showed that 75% of people like chocolate chip, 50% like peanut butter, and only 3% like coconut. The use of this information makes sense in which of the following scenarios? A. The school cafeteria decides to make 3 coconut cookies for each 100 students who buy lunch, even though it puts them over budget for desserts. B. A national cookie company is thinking of changing the recipe in their peanut butter cookies. C. A national cookie company decides to spend $5 million in advertising to convince people to eat coconut cookies. D. Tom is about to open a small bakery and is using the result of the sampling to decide what kind of cookies to offer.

Answers

Answer:

D. Tom is about to open a small bakery and is using the result of the sampling to decide what kind of cookies to offer.

Step-by-step explanation:

I believe that is correct....

Find the perimeter of this figure. (Image down below)

Answers

Answer:

The answer is

44 ft

Step-by-step explanation:

The figure above is a rectangle

To find the perimeter of a rectangle we use the formula

Perimeter of a rectangle = 2l + 2w

where

l is the length of the rectangle

w is the width of the rectangle

From the question

length = 14 ft

width = 8ft

Substituting the values into the above formula we have

Perimeter = 2(14) + 2(8)

= 28 + 16

We have the final answer as

Perimeter = 44 ft

Hope this helps you

Since perimeter is adding all the sides together it would be (8×2 = 16) and (14×2 = 28 ) so 28+16=44 ^^ I agree with the answer above :)

Compute the range and interquartile range for the data collected for boys and girls. Describe their differences in detail using specific terms of spread. (4 points)

Answers

Answer:

The measure of central tendency, mean and median are approximately equal for the boys indicating that the data of the boys is more evenly spread while standard deviation of the girls data is less than those of the boys indicating that the data for the girls is less widely spread.

Step-by-step explanation:

The given data are;

,             1       2       3      4     5      6       7    8      9      10

Girls,    50    32     15    56   81    50     18    81    22    55

Boys,   75     41     25    22    7     0      43    12    45    70

Sorting the data gives;

Girls,     15,   18,     22,   32,  50,   50,   55,  56,    81,  81

Boys,     0,     7,     12,     22, 25,    41,    43, 45,     70, 75

For the even numbered sample data size, the first quartile, Q₁ is found by sharing the data into two and finding the median of the left half which  gives;

10/2 = 5 on each half

The first quartile, Q₁, is the median of the left 5 data points which is the 3rd data point = 22 for girls and 12 for boys

The third quartile, Q₃, is found in similar method to be the 8th data point which is 56 for girls and 45 for boys

The median = 50 for girls and 33 for boys

Therefore, the interquartile ranges are;

IQR = 56 - 22 = 34 for girls, 45 - 12 = 33 for boys

We check for outliers.

Q₁ - 1.5×IQR = 22 - 1.5*34 = -29

Q₃ + 1.5×IQR = 56 + 1.5*34 = 107

We check the mean of both data samples as follows;

Average for the girls = 46

Average for the boys = 34

Standard deviation for girls = 23.99

Standard deviation for girls = 25.43

Therefore, the measure of central tendency is more accurate for the boys indicating that the data of the boys is more evenly spread while the data for the girls is less widely spread.

it’s either A or B but idk, i’ll give brainliest :)

Answers

Answer:

Actually it's A.

Step-by-step explanation:

Find the measure Of PR

Answers

Answer:

18

Step-by-step explanation:

The formula for this is a x b = c x d where a is SR, b is FR, c is QR and d is PR. This will give us the equation 9(16) = 8(5x + 8); 144 = 40x + 64; 80 = 40x; x = 2. Now, plug this into PR which will be 5(2)+8 = 18. This is your answer.

Each day that a library book is kept past its due date a 30 dollar fee is charged at midnight which ordered pair is a viable solution if x represents the number of days that a library book is late and y represents the total fee

Answers

THIS IS THE COMPLETE QUESTION BELOW;

Each day that a library book is kept past its due date, a $0.30 fee is charged at midnight. Which ordered pair is a viable solution if x represents the number of days that a library book is late and y represents the total fee?

Answers:

(–3, –0.90)

(–2.5, –0.75)

(4.5, 1.35)

(8, 2.40)

Answer

(8, 2.40)

Step by step Explanation

✓We can denote the number of days library book is late = X,

✓We can denote the the total fee = Y.

We were told $0.30 fee is charged at midnight.

Then for lateness for just 1day,the charged fees= 1day ×

$0.30

For X number of days the charged fees= Xday ×$0.30

Therefore, total charge for lateness for X number of days late = Y.

Then can be expressed as

Y= 0.30 * X...............eqn(1)

We can now test the option one after the other

FIRST OPTION (-3, -0.9)

Here we should know the number of days cannot be negative so there is no need of testing in the equation (1)

SECOND OPTION (-2.5, -0.75)

Here we should know the number of days cannot be negative so there is no need of testing in the equation (1)

THIRD OPTION(4.5, 1.35)

here the number of days will definitely be a whole number not 4.5, it's either

charge for 4 days or 5 days.

FORTH OPTION (8, 2.40)

this should be correct because the number of days is whole number and not negative, then if we test it from our equation it satisfy the equation too

Y= 0.30 * X...............eqn(1)

Y= 0.30 * X

2.40= 0.30 * 8

2.40 = 2.40.

Therefore, (8, 2.40) is the answer

SOMEONE PLZ HELP ASAP !!!!!!

Answers

Answer:

a = 53.13

b = 42.76

c= 0

Step-by-step explanation:

as the question said i was supposed to use a calculator which i did

A 3 L bottle of oil costs $36 and contains 12 cups. Dinesh puts 1 cup of oil, 10 garlic gloves and 1 cup of
lemon juice in each batch of hummus recipe that he makes. Dinesh makes 5 batches of hummus.
What is the total cost of oil that he uses in the 5 batches of his recipe?

Answers

The correct answer is $15

Explanation:

The first step is to determine the total of oil that was used for the 5 batches. To find this, you just need to multiply the amount of oil used for one batch by the total batches.

1 cup of oil per batch x 5 batches = 5 cups of oil

This means, in the 5 batches the oil Dinesh used was 5 cups of oil. Additionally, you know the total of cups in the bottle of oil is 12 cups, and these 12 cups or total costs $36. Now to find what is the cost of the 5 cups use the rule of three and cross multiplication.

12 cups of oil = $36   1. Write the values

5 cups of oil  = x      

12 x = 180           2.  Cross multiply this means 36x 5 and 12 multipy by x

x = 180 ÷ 12        3. Solve the equation

x=  15 - Cost for 5 cups used in the batches

II
Initial Knowledge
This morning, Leila's car had 19.79 gallons of fuel. Now, 2.8 gallons are left. How much fuel did Leila use?

Answers

Answer:

[tex]19.79 - 2.8 = 16.99gallons[/tex]

Can someone help!!! And explain please

Answers

Answer:

400(π+2) feet square

Step-by-step explanation:

let x be the diagonal of the cage=40√2 at the same time it is the radius of the circle ( the tiger can go in circle)

but since the cage is part of the circle and not full turn πr²/8

area of the circleπr²+ half area square

(π(40√2)²)/8 +40²/2

3200π/8+1600/2

400π+800

400(π+2) feet square

Other Questions
A 137 kg horizontal platform is a uniform disk of radius 1.53 m and can rotate about the vertical axis through its center. A 68.7 kg person stands on the platform at a distance of 1.19 m from the center, and a 25.9 kg dog sits on the platform near the person 1.45 m from the center. Find the moment of inertia of this system, consisting of the platform and its population, with respect to the axis. In one paragraph, explain why leaders of the women's rights movement decided to model the Declaration of Sentiments after the Declaration of Independence. Which of the following was a distinctive outcome of the Agricultural Revolution? Determine the value(s) for which the rational expression 2x^2/6x is undefined. If there's more than one value, list them separated by a comma, e.g. x=2,3. Which statement best describes the meter? The meter is free, imitating the flow of conversational speech. The meter is free, making the poem unpredictable. The meter is fixed, and every syllable is stressed. The meter is fixed, drawing attention to the end rhyme. For this year's fundraiser, students at a certain school who sell at least 75 magazine subscriptions win a prize. If the fourth grade students at this school sell an average (arithmetic mean) of 47 subscriptions per student, the sales are normally distributed, and have a standard deviation of 14, then approximately what percent of the fourth grade students receive a prize Pls help for the writings above to be the brainlist . Which of the following peripheral devices can be used for both input and output? mouse touch screen on a tablet computer printer CPU Solve x*2+5x+6=5x+10 Which of the following goals of a performance evaluation system is accomplished when the company's actual results are compared to industry standards? A) Benchmarking B) Motivating unit managers C) Promoting goal congruence D) Providing feedback Eric left high school to work in a factory where he has been for the last 9 years. He married at 19 and has two children. He is unhappy and cynical. He doesn't like working hard to make purchase decisions so he waits until a product is easy to find before he buys. A. EA B. EM C. ID. LE. LM rancis Inc.'s stock has a required rate of return of 10.25%, and it sells for $87.50 per share. The dividend is expected to grow at a constant rate of 6.00% per year. What is the expected year-end dividend, D1 A spinner has 10 equally sized sections, 5 of which are gray and 5 of which are blue. The spinner is spun twice. What is the probability that the first spin lands on gray and the second spin lands on blue? Write your answer as a fraction in the simplest form. 2. Which sentence has captions broken at logical places?O-l live right by the mountains, and soI only go to the beach with myfriends during the summer.O I live right by the mountains,And so I only go to the beach withmy friends during the summer.Ollive right by the mountains, andsol only go to thebeach with my friends duringthe summer.. I live right by the mountainsand so I only go to the beachwith my friends during the summer. Portia Grant is an employee who is paid monthly. For the month of January of the current year, she earned a total of 8,588. The FICA tax for social security is 6.2% of the first $118,500 of employee earnings each calendar year and the FICA tax rate for Medicare is 1.45% of all earnings. The FUTA tax rate of 0.6% and the SUTA tax rate of 5.4% are applied to the first $7,000 of an employee's pay. The amount of federal income tax withheld from her earnings was $1,424.97. Her net pay for the month is: (Round your intermediate calculations to two decimal places.) a) $7,038.50 b) $5,141.03 c) $6,072.04 d) $6,566.00 e) $6,506.04 2. Why do you think the plan to get the Confederates tosurrender was named after an Anaconda (a large blacksnake)? f(x) = -3x + 7What is f (0)? A 50.0 L cylinder of oxygen gas is stored at 150. atm. What volume would the oxygen gas occupy if the cylinder were opened into a hot air balloon (completely deflated) until the final pressure is 735 torr Can podocyte cells in the Bowmann capsule attach to any other basement membrane other than the glomerular basement membrane? That is, it can itself have a separate layer of base membrane? Assume that your parents wanted to have a 170,000 saved for college by your 18th birthday and they started saving on your first birthday. They saved the same amount each year on your birthday and earned 6.5% per year on their investmenets.Required:a. How much would they have to save each year to reach their goal?b. If they think you will take five years instead of four to graduate to graduate and decide to have $140,000 saved just in case, how much more would they have to save each year to reach their new goal?